3
$\begingroup$

For the absolute value $|C|=(C^*C)^\frac{1}{2}$ and the Hilbert-Schmidt norm $\parallel C\parallel_{HS}=(trC^*C)^\frac{1}{2}$ of the operator $C$. The following inequality is shown by Araki et al in 'An Inequality for Hilbert-Schmidt Norm, Commun. Math. Phys. 81, 89-96 (1981)'

For any two bounded linear operators $A$ and $B$ on a Hilbert space $\mathbb{H}$, $$\parallel |A|-|B| \parallel_{HS}\le \sqrt{2}\parallel A-B \parallel_{HS},$$ and the factor $\sqrt{2}$ is best possible.

What is the best constant factor $c$ such that $$\parallel |A|+|B| \parallel_{HS}\ge c\parallel A+B \parallel_{HS}?$$

$\endgroup$
5
  • $\begingroup$ infinity. Take A to be a non-zero positive operator and B = -A. Incidentally, either you meant to say "finite dimensional Hilbert space" or you meant "two Hilbert-Schmidt linear operators" since not all bounded linear operators on a Hilbert space have a Hilbert-Schmidt norm. $\endgroup$ Jun 21, 2010 at 7:40
  • $\begingroup$ Andrew, that doesn't work - the inequality is "the other way round" from what your comment seems to say. $\endgroup$
    – Yemon Choi
    Jun 21, 2010 at 7:47
  • $\begingroup$ @Stacey: $A, B$ are arbitrary. In this case, I actually want to determine the maximum of $c$ such that the inequality holds for arbitrary bounded linear operators $A, B$. $\endgroup$
    – Russel
    Jun 21, 2010 at 7:53
  • $\begingroup$ @Yemon: Whoops! Just goes to show that "best is in the eye of the beholder". @Russel: The identity operator on an infinite dimensional Hilbert space does not have a Hilbert-Schmidt norm. So for your question to make sense, you need to include some conditions to exclude this case. $\endgroup$ Jun 21, 2010 at 8:02
  • $\begingroup$ @Stacey: The condition here is the same as in the paper by H.Araki. We allow an operator with norm infinity. $\endgroup$
    – Russel
    Jun 21, 2010 at 8:13

1 Answer 1

1
$\begingroup$

edit: there is a problem with this, see the comments; I'm leaving this up for the moment in case it helps someone else come along to write a better answer.


I'm pretty sure that the inequality $$ \vert {\rm tr}(A^*B) \vert \leq {\rm tr}( |A| |B| ) $$ holds whenever $A$ and $B$ are Hilbert-Schmidt, just by using the polar decompositions of $A$ and $B$.

If this is the case, then we'd have

$$ \eqalign{ {\rm tr} ((A+B)^*(A+B)) & = {\rm tr}(A^*A) + {\rm tr}(A^*B) + {\rm tr}(B^*A) + {\rm tr}(B^*B) \\ & \leq {\rm tr}(\vert A\vert^2) + {\rm tr}(\vert A\vert \vert B\vert) + {\rm tr}(\vert B\vert \vert A\vert) + {\rm tr}(\vert B\vert^2) = {\rm tr}((\vert A\vert + \vert B\vert)^2) } $$ which implies that you can get away with $c=1$. Taking $A=B$ to be positive shows that this is sharp.

$\endgroup$
3
  • $\begingroup$ I'm not seeing, right now, why $|tr(A^*B)|\leq tr(|A| |B|)$. Can you give a hint... $\endgroup$ Jun 21, 2010 at 8:19
  • 1
    $\begingroup$ It is true $|tr(AB)|\leq tr(|AB|)$, but it is not true in general $tr(|AB|)\leq tr(|A| |B|)$. $\endgroup$
    – Russel
    Jun 21, 2010 at 8:29
  • $\begingroup$ Hmm, I may have been careless here: I was just thinking of putting $A= U \vert A\vert$ and $B=V \vert B\vert$ for partial isometries $U$ and $V$, then ${\rm tr}(A^*B) = {\rm tr} (\vert A\vert U^*V \vert B\vert)$ -- but then I've overlooked the fact that $\vert A\vert \vert B\vert$ might not be positive... Will have to think about this more! $\endgroup$
    – Yemon Choi
    Jun 21, 2010 at 8:33

Your Answer

By clicking “Post Your Answer”, you agree to our terms of service and acknowledge you have read our privacy policy.

Not the answer you're looking for? Browse other questions tagged or ask your own question.